Prove that $ int _0^inftyfracsin xxdx=fracpi2$ Announcing the arrival of Valued Associate #679: Cesar Manara Planned maintenance scheduled April 17/18, 2019 at 00:00UTC (8:00pm US/Eastern)$sum_n=-infty^inftyfrac1(n+alpha)^2=fracpi^2(sin pi alpha)^2$?Show $lim_Ntoinftyint_0^pileft(frac1sinfracx2-frac2xright)sinleft((N+frac12)xright)dx=0$Show that Fourier transformation is differentiable if $int|xf(x)|,dlambda<infty$Riemann Lebesgue Lemma Clarification$lim_lambda to infty int^b_0 f(t) fracsin(lambda t)t $The improper integrals of $fracsin tt$ and $left|fracsin ttright|$Calculate limits of integralsAlternate proof of Dirichlet integral $fracsin(x)x$.Riemann-Lebesgue Lemma And Integral Having $sin(cdot)$Abel means of function f when having a jump discontinuity

Is it ethical to give a final exam after the professor has quit before teaching the remaining chapters of the course?

Why are there no cargo aircraft with "flying wing" design?

Why are both D and D# fitting into my E minor key?

When the Haste spell ends on a creature, do attackers have advantage against that creature?

Extracting terms with certain heads in a function

Do wooden building fires get hotter than 600°C?

Fundamental Solution of the Pell Equation

Has negative voting ever been officially implemented in elections, or seriously proposed, or even studied?

How could we fake a moon landing now?

Where are Serre’s lectures at Collège de France to be found?

Denied boarding although I have proper visa and documentation. To whom should I make a complaint?

What is the longest distance a player character can jump in one leap?

What causes the direction of lightning flashes?

What do you call the main part of a joke?

Is it cost-effective to upgrade an old-ish Giant Escape R3 commuter bike with entry-level branded parts (wheels, drivetrain)?

Most bit efficient text communication method?

Generate an RGB colour grid

Wu formula for manifolds with boundary

An adverb for when you're not exaggerating

Is safe to use va_start macro with this as parameter?

How to react to hostile behavior from a senior developer?

What are the out-of-universe reasons for the references to Toby Maguire-era Spider-Man in ITSV

What does the "x" in "x86" represent?

Using audio cues to encourage good posture



Prove that $ int _0^inftyfracsin xxdx=fracpi2$



Announcing the arrival of Valued Associate #679: Cesar Manara
Planned maintenance scheduled April 17/18, 2019 at 00:00UTC (8:00pm US/Eastern)$sum_n=-infty^inftyfrac1(n+alpha)^2=fracpi^2(sin pi alpha)^2$?Show $lim_Ntoinftyint_0^pileft(frac1sinfracx2-frac2xright)sinleft((N+frac12)xright)dx=0$Show that Fourier transformation is differentiable if $int|xf(x)|,dlambda<infty$Riemann Lebesgue Lemma Clarification$lim_lambda to infty int^b_0 f(t) fracsin(lambda t)t $The improper integrals of $fracsin tt$ and $left|fracsin ttright|$Calculate limits of integralsAlternate proof of Dirichlet integral $fracsin(x)x$.Riemann-Lebesgue Lemma And Integral Having $sin(cdot)$Abel means of function f when having a jump discontinuity










1












$begingroup$


Prove
$$int_0^inftyfracsin xxdx=fracpi2$$
Start with the fact that the integral of $D_N(theta)$ equals $2pi$, and note that the difference $$frac1sin(theta/2)-frac2theta$$
is continuous on $[-pi,pi]$. Apply the Riemann-Lebesgue lemma.
$$D_N(x)=sum_-N^Ne^inx=fracsin((N+1/2)x)sin(x/2)$$
Now I can only think to apply the Riemann-Lebesgue lemma to the function
$$frac1sin(theta/2)-frac2theta$$
To get :
$$lim_nto inftyfrac12piint_-pi^pi(frace^-inthetasin(theta/2)-frac2e^-inthetatheta)dthetato0$$But I don't know the steps to prove this.










share|cite|improve this question











$endgroup$











  • $begingroup$
    It appears you're trying to follow a specific argument for the integral here. In order for the rest of us to understand what you're doing, we need more context about that argument. For example, what is $D_N$?
    $endgroup$
    – jmerry
    Mar 27 at 4:57










  • $begingroup$
    you have a limit as $n to infty$ on the LHS and an $n$ on the RHS
    $endgroup$
    – mathworker21
    Mar 27 at 4:58










  • $begingroup$
    On the last line, both integrals are badly non-convergent.
    $endgroup$
    – Lord Shark the Unknown
    Mar 27 at 5:07










  • $begingroup$
    I changed it according to the form of Riemann-Lebesgue lemma.
    $endgroup$
    – John He
    Mar 27 at 5:13










  • $begingroup$
    @jmerry - If I had to guess at a glance, it probably is tied to the Dirichlet kernel somehow or another
    $endgroup$
    – Eevee Trainer
    Mar 27 at 5:14















1












$begingroup$


Prove
$$int_0^inftyfracsin xxdx=fracpi2$$
Start with the fact that the integral of $D_N(theta)$ equals $2pi$, and note that the difference $$frac1sin(theta/2)-frac2theta$$
is continuous on $[-pi,pi]$. Apply the Riemann-Lebesgue lemma.
$$D_N(x)=sum_-N^Ne^inx=fracsin((N+1/2)x)sin(x/2)$$
Now I can only think to apply the Riemann-Lebesgue lemma to the function
$$frac1sin(theta/2)-frac2theta$$
To get :
$$lim_nto inftyfrac12piint_-pi^pi(frace^-inthetasin(theta/2)-frac2e^-inthetatheta)dthetato0$$But I don't know the steps to prove this.










share|cite|improve this question











$endgroup$











  • $begingroup$
    It appears you're trying to follow a specific argument for the integral here. In order for the rest of us to understand what you're doing, we need more context about that argument. For example, what is $D_N$?
    $endgroup$
    – jmerry
    Mar 27 at 4:57










  • $begingroup$
    you have a limit as $n to infty$ on the LHS and an $n$ on the RHS
    $endgroup$
    – mathworker21
    Mar 27 at 4:58










  • $begingroup$
    On the last line, both integrals are badly non-convergent.
    $endgroup$
    – Lord Shark the Unknown
    Mar 27 at 5:07










  • $begingroup$
    I changed it according to the form of Riemann-Lebesgue lemma.
    $endgroup$
    – John He
    Mar 27 at 5:13










  • $begingroup$
    @jmerry - If I had to guess at a glance, it probably is tied to the Dirichlet kernel somehow or another
    $endgroup$
    – Eevee Trainer
    Mar 27 at 5:14













1












1








1





$begingroup$


Prove
$$int_0^inftyfracsin xxdx=fracpi2$$
Start with the fact that the integral of $D_N(theta)$ equals $2pi$, and note that the difference $$frac1sin(theta/2)-frac2theta$$
is continuous on $[-pi,pi]$. Apply the Riemann-Lebesgue lemma.
$$D_N(x)=sum_-N^Ne^inx=fracsin((N+1/2)x)sin(x/2)$$
Now I can only think to apply the Riemann-Lebesgue lemma to the function
$$frac1sin(theta/2)-frac2theta$$
To get :
$$lim_nto inftyfrac12piint_-pi^pi(frace^-inthetasin(theta/2)-frac2e^-inthetatheta)dthetato0$$But I don't know the steps to prove this.










share|cite|improve this question











$endgroup$




Prove
$$int_0^inftyfracsin xxdx=fracpi2$$
Start with the fact that the integral of $D_N(theta)$ equals $2pi$, and note that the difference $$frac1sin(theta/2)-frac2theta$$
is continuous on $[-pi,pi]$. Apply the Riemann-Lebesgue lemma.
$$D_N(x)=sum_-N^Ne^inx=fracsin((N+1/2)x)sin(x/2)$$
Now I can only think to apply the Riemann-Lebesgue lemma to the function
$$frac1sin(theta/2)-frac2theta$$
To get :
$$lim_nto inftyfrac12piint_-pi^pi(frace^-inthetasin(theta/2)-frac2e^-inthetatheta)dthetato0$$But I don't know the steps to prove this.







fourier-analysis






share|cite|improve this question















share|cite|improve this question













share|cite|improve this question




share|cite|improve this question








edited Mar 27 at 5:11







John He

















asked Mar 27 at 4:52









John HeJohn He

668




668











  • $begingroup$
    It appears you're trying to follow a specific argument for the integral here. In order for the rest of us to understand what you're doing, we need more context about that argument. For example, what is $D_N$?
    $endgroup$
    – jmerry
    Mar 27 at 4:57










  • $begingroup$
    you have a limit as $n to infty$ on the LHS and an $n$ on the RHS
    $endgroup$
    – mathworker21
    Mar 27 at 4:58










  • $begingroup$
    On the last line, both integrals are badly non-convergent.
    $endgroup$
    – Lord Shark the Unknown
    Mar 27 at 5:07










  • $begingroup$
    I changed it according to the form of Riemann-Lebesgue lemma.
    $endgroup$
    – John He
    Mar 27 at 5:13










  • $begingroup$
    @jmerry - If I had to guess at a glance, it probably is tied to the Dirichlet kernel somehow or another
    $endgroup$
    – Eevee Trainer
    Mar 27 at 5:14
















  • $begingroup$
    It appears you're trying to follow a specific argument for the integral here. In order for the rest of us to understand what you're doing, we need more context about that argument. For example, what is $D_N$?
    $endgroup$
    – jmerry
    Mar 27 at 4:57










  • $begingroup$
    you have a limit as $n to infty$ on the LHS and an $n$ on the RHS
    $endgroup$
    – mathworker21
    Mar 27 at 4:58










  • $begingroup$
    On the last line, both integrals are badly non-convergent.
    $endgroup$
    – Lord Shark the Unknown
    Mar 27 at 5:07










  • $begingroup$
    I changed it according to the form of Riemann-Lebesgue lemma.
    $endgroup$
    – John He
    Mar 27 at 5:13










  • $begingroup$
    @jmerry - If I had to guess at a glance, it probably is tied to the Dirichlet kernel somehow or another
    $endgroup$
    – Eevee Trainer
    Mar 27 at 5:14















$begingroup$
It appears you're trying to follow a specific argument for the integral here. In order for the rest of us to understand what you're doing, we need more context about that argument. For example, what is $D_N$?
$endgroup$
– jmerry
Mar 27 at 4:57




$begingroup$
It appears you're trying to follow a specific argument for the integral here. In order for the rest of us to understand what you're doing, we need more context about that argument. For example, what is $D_N$?
$endgroup$
– jmerry
Mar 27 at 4:57












$begingroup$
you have a limit as $n to infty$ on the LHS and an $n$ on the RHS
$endgroup$
– mathworker21
Mar 27 at 4:58




$begingroup$
you have a limit as $n to infty$ on the LHS and an $n$ on the RHS
$endgroup$
– mathworker21
Mar 27 at 4:58












$begingroup$
On the last line, both integrals are badly non-convergent.
$endgroup$
– Lord Shark the Unknown
Mar 27 at 5:07




$begingroup$
On the last line, both integrals are badly non-convergent.
$endgroup$
– Lord Shark the Unknown
Mar 27 at 5:07












$begingroup$
I changed it according to the form of Riemann-Lebesgue lemma.
$endgroup$
– John He
Mar 27 at 5:13




$begingroup$
I changed it according to the form of Riemann-Lebesgue lemma.
$endgroup$
– John He
Mar 27 at 5:13












$begingroup$
@jmerry - If I had to guess at a glance, it probably is tied to the Dirichlet kernel somehow or another
$endgroup$
– Eevee Trainer
Mar 27 at 5:14




$begingroup$
@jmerry - If I had to guess at a glance, it probably is tied to the Dirichlet kernel somehow or another
$endgroup$
– Eevee Trainer
Mar 27 at 5:14










2 Answers
2






active

oldest

votes


















1












$begingroup$

Note that
$$int_-pi^pi D_N(t),dt=2pi.$$
Use that
$$int_-pi^pisin((N+1/2)t)left(frac1sin t/2-frac 2tright),dtto0$$
as $Ntoinfty$ (Riemann-Lebesgue) to deduce that
$$lim_Ntoinftyint_-pi^pifracsin((N+1/2)t)t,dx=pi.$$






share|cite|improve this answer









$endgroup$












  • $begingroup$
    Thank you. I did It again with My original path, and I find It's essentially the same thing.
    $endgroup$
    – John He
    Mar 27 at 5:30


















0












$begingroup$

Consider the Laplace transform
$$
F(s)=int_0^inftye^-sxfracsin xxdx.
$$

Your integral is $lim_sdownarrow 0F(s)$. For $s > 0$,
$$
F'(s)=int_0^inftye^-sxsin xdx \
= Im int_0^inftye^-sxe^ixdx \
= Im int_0^inftye^-x(s+i)dx \
= Im frac1s+i=Imfracs-i1+s^2 \=-frac11+s^2
$$

Therefore $F(s)=-tan^-1(s)+C$ for some constant $C$. Because $F(infty)=0$, then $C=fracpi2$. So
$$
int_0^inftye^-sxfracsin xxdx = fracpi2-tan^-1(s).
$$

Letting $sdownarrow 0$ gives
$$
int_0^inftyfracsin xxdx=fracpi2.
$$






share|cite|improve this answer









$endgroup$













    Your Answer








    StackExchange.ready(function()
    var channelOptions =
    tags: "".split(" "),
    id: "69"
    ;
    initTagRenderer("".split(" "), "".split(" "), channelOptions);

    StackExchange.using("externalEditor", function()
    // Have to fire editor after snippets, if snippets enabled
    if (StackExchange.settings.snippets.snippetsEnabled)
    StackExchange.using("snippets", function()
    createEditor();
    );

    else
    createEditor();

    );

    function createEditor()
    StackExchange.prepareEditor(
    heartbeatType: 'answer',
    autoActivateHeartbeat: false,
    convertImagesToLinks: true,
    noModals: true,
    showLowRepImageUploadWarning: true,
    reputationToPostImages: 10,
    bindNavPrevention: true,
    postfix: "",
    imageUploader:
    brandingHtml: "Powered by u003ca class="icon-imgur-white" href="https://imgur.com/"u003eu003c/au003e",
    contentPolicyHtml: "User contributions licensed under u003ca href="https://creativecommons.org/licenses/by-sa/3.0/"u003ecc by-sa 3.0 with attribution requiredu003c/au003e u003ca href="https://stackoverflow.com/legal/content-policy"u003e(content policy)u003c/au003e",
    allowUrls: true
    ,
    noCode: true, onDemand: true,
    discardSelector: ".discard-answer"
    ,immediatelyShowMarkdownHelp:true
    );



    );













    draft saved

    draft discarded


















    StackExchange.ready(
    function ()
    StackExchange.openid.initPostLogin('.new-post-login', 'https%3a%2f%2fmath.stackexchange.com%2fquestions%2f3164117%2fprove-that-int-0-infty-frac-sin-xxdx-frac-pi2%23new-answer', 'question_page');

    );

    Post as a guest















    Required, but never shown

























    2 Answers
    2






    active

    oldest

    votes








    2 Answers
    2






    active

    oldest

    votes









    active

    oldest

    votes






    active

    oldest

    votes









    1












    $begingroup$

    Note that
    $$int_-pi^pi D_N(t),dt=2pi.$$
    Use that
    $$int_-pi^pisin((N+1/2)t)left(frac1sin t/2-frac 2tright),dtto0$$
    as $Ntoinfty$ (Riemann-Lebesgue) to deduce that
    $$lim_Ntoinftyint_-pi^pifracsin((N+1/2)t)t,dx=pi.$$






    share|cite|improve this answer









    $endgroup$












    • $begingroup$
      Thank you. I did It again with My original path, and I find It's essentially the same thing.
      $endgroup$
      – John He
      Mar 27 at 5:30















    1












    $begingroup$

    Note that
    $$int_-pi^pi D_N(t),dt=2pi.$$
    Use that
    $$int_-pi^pisin((N+1/2)t)left(frac1sin t/2-frac 2tright),dtto0$$
    as $Ntoinfty$ (Riemann-Lebesgue) to deduce that
    $$lim_Ntoinftyint_-pi^pifracsin((N+1/2)t)t,dx=pi.$$






    share|cite|improve this answer









    $endgroup$












    • $begingroup$
      Thank you. I did It again with My original path, and I find It's essentially the same thing.
      $endgroup$
      – John He
      Mar 27 at 5:30













    1












    1








    1





    $begingroup$

    Note that
    $$int_-pi^pi D_N(t),dt=2pi.$$
    Use that
    $$int_-pi^pisin((N+1/2)t)left(frac1sin t/2-frac 2tright),dtto0$$
    as $Ntoinfty$ (Riemann-Lebesgue) to deduce that
    $$lim_Ntoinftyint_-pi^pifracsin((N+1/2)t)t,dx=pi.$$






    share|cite|improve this answer









    $endgroup$



    Note that
    $$int_-pi^pi D_N(t),dt=2pi.$$
    Use that
    $$int_-pi^pisin((N+1/2)t)left(frac1sin t/2-frac 2tright),dtto0$$
    as $Ntoinfty$ (Riemann-Lebesgue) to deduce that
    $$lim_Ntoinftyint_-pi^pifracsin((N+1/2)t)t,dx=pi.$$







    share|cite|improve this answer












    share|cite|improve this answer



    share|cite|improve this answer










    answered Mar 27 at 5:16









    Lord Shark the UnknownLord Shark the Unknown

    109k1163136




    109k1163136











    • $begingroup$
      Thank you. I did It again with My original path, and I find It's essentially the same thing.
      $endgroup$
      – John He
      Mar 27 at 5:30
















    • $begingroup$
      Thank you. I did It again with My original path, and I find It's essentially the same thing.
      $endgroup$
      – John He
      Mar 27 at 5:30















    $begingroup$
    Thank you. I did It again with My original path, and I find It's essentially the same thing.
    $endgroup$
    – John He
    Mar 27 at 5:30




    $begingroup$
    Thank you. I did It again with My original path, and I find It's essentially the same thing.
    $endgroup$
    – John He
    Mar 27 at 5:30











    0












    $begingroup$

    Consider the Laplace transform
    $$
    F(s)=int_0^inftye^-sxfracsin xxdx.
    $$

    Your integral is $lim_sdownarrow 0F(s)$. For $s > 0$,
    $$
    F'(s)=int_0^inftye^-sxsin xdx \
    = Im int_0^inftye^-sxe^ixdx \
    = Im int_0^inftye^-x(s+i)dx \
    = Im frac1s+i=Imfracs-i1+s^2 \=-frac11+s^2
    $$

    Therefore $F(s)=-tan^-1(s)+C$ for some constant $C$. Because $F(infty)=0$, then $C=fracpi2$. So
    $$
    int_0^inftye^-sxfracsin xxdx = fracpi2-tan^-1(s).
    $$

    Letting $sdownarrow 0$ gives
    $$
    int_0^inftyfracsin xxdx=fracpi2.
    $$






    share|cite|improve this answer









    $endgroup$

















      0












      $begingroup$

      Consider the Laplace transform
      $$
      F(s)=int_0^inftye^-sxfracsin xxdx.
      $$

      Your integral is $lim_sdownarrow 0F(s)$. For $s > 0$,
      $$
      F'(s)=int_0^inftye^-sxsin xdx \
      = Im int_0^inftye^-sxe^ixdx \
      = Im int_0^inftye^-x(s+i)dx \
      = Im frac1s+i=Imfracs-i1+s^2 \=-frac11+s^2
      $$

      Therefore $F(s)=-tan^-1(s)+C$ for some constant $C$. Because $F(infty)=0$, then $C=fracpi2$. So
      $$
      int_0^inftye^-sxfracsin xxdx = fracpi2-tan^-1(s).
      $$

      Letting $sdownarrow 0$ gives
      $$
      int_0^inftyfracsin xxdx=fracpi2.
      $$






      share|cite|improve this answer









      $endgroup$















        0












        0








        0





        $begingroup$

        Consider the Laplace transform
        $$
        F(s)=int_0^inftye^-sxfracsin xxdx.
        $$

        Your integral is $lim_sdownarrow 0F(s)$. For $s > 0$,
        $$
        F'(s)=int_0^inftye^-sxsin xdx \
        = Im int_0^inftye^-sxe^ixdx \
        = Im int_0^inftye^-x(s+i)dx \
        = Im frac1s+i=Imfracs-i1+s^2 \=-frac11+s^2
        $$

        Therefore $F(s)=-tan^-1(s)+C$ for some constant $C$. Because $F(infty)=0$, then $C=fracpi2$. So
        $$
        int_0^inftye^-sxfracsin xxdx = fracpi2-tan^-1(s).
        $$

        Letting $sdownarrow 0$ gives
        $$
        int_0^inftyfracsin xxdx=fracpi2.
        $$






        share|cite|improve this answer









        $endgroup$



        Consider the Laplace transform
        $$
        F(s)=int_0^inftye^-sxfracsin xxdx.
        $$

        Your integral is $lim_sdownarrow 0F(s)$. For $s > 0$,
        $$
        F'(s)=int_0^inftye^-sxsin xdx \
        = Im int_0^inftye^-sxe^ixdx \
        = Im int_0^inftye^-x(s+i)dx \
        = Im frac1s+i=Imfracs-i1+s^2 \=-frac11+s^2
        $$

        Therefore $F(s)=-tan^-1(s)+C$ for some constant $C$. Because $F(infty)=0$, then $C=fracpi2$. So
        $$
        int_0^inftye^-sxfracsin xxdx = fracpi2-tan^-1(s).
        $$

        Letting $sdownarrow 0$ gives
        $$
        int_0^inftyfracsin xxdx=fracpi2.
        $$







        share|cite|improve this answer












        share|cite|improve this answer



        share|cite|improve this answer










        answered Mar 29 at 2:40









        DisintegratingByPartsDisintegratingByParts

        60.5k42681




        60.5k42681



























            draft saved

            draft discarded
















































            Thanks for contributing an answer to Mathematics Stack Exchange!


            • Please be sure to answer the question. Provide details and share your research!

            But avoid


            • Asking for help, clarification, or responding to other answers.

            • Making statements based on opinion; back them up with references or personal experience.

            Use MathJax to format equations. MathJax reference.


            To learn more, see our tips on writing great answers.




            draft saved


            draft discarded














            StackExchange.ready(
            function ()
            StackExchange.openid.initPostLogin('.new-post-login', 'https%3a%2f%2fmath.stackexchange.com%2fquestions%2f3164117%2fprove-that-int-0-infty-frac-sin-xxdx-frac-pi2%23new-answer', 'question_page');

            );

            Post as a guest















            Required, but never shown





















































            Required, but never shown














            Required, but never shown












            Required, but never shown







            Required, but never shown

































            Required, but never shown














            Required, but never shown












            Required, but never shown







            Required, but never shown







            Popular posts from this blog

            How should I support this large drywall patch? Planned maintenance scheduled April 23, 2019 at 00:00UTC (8:00pm US/Eastern) Announcing the arrival of Valued Associate #679: Cesar Manara Unicorn Meta Zoo #1: Why another podcast?How do I cover large gaps in drywall?How do I keep drywall around a patch from crumbling?Can I glue a second layer of drywall?How to patch long strip on drywall?Large drywall patch: how to avoid bulging seams?Drywall Mesh Patch vs. Bulge? To remove or not to remove?How to fix this drywall job?Prep drywall before backsplashWhat's the best way to fix this horrible drywall patch job?Drywall patching using 3M Patch Plus Primer

            Lowndes Grove History Architecture References Navigation menu32°48′6″N 79°57′58″W / 32.80167°N 79.96611°W / 32.80167; -79.9661132°48′6″N 79°57′58″W / 32.80167°N 79.96611°W / 32.80167; -79.9661178002500"National Register Information System"Historic houses of South Carolina"Lowndes Grove""+32° 48' 6.00", −79° 57' 58.00""Lowndes Grove, Charleston County (260 St. Margaret St., Charleston)""Lowndes Grove"The Charleston ExpositionIt Happened in South Carolina"Lowndes Grove (House), Saint Margaret Street & Sixth Avenue, Charleston, Charleston County, SC(Photographs)"Plantations of the Carolina Low Countrye

            random experiment with two different functions on unit interval Announcing the arrival of Valued Associate #679: Cesar Manara Planned maintenance scheduled April 23, 2019 at 00:00UTC (8:00pm US/Eastern)Random variable and probability space notionsRandom Walk with EdgesFinding functions where the increase over a random interval is Poisson distributedNumber of days until dayCan an observed event in fact be of zero probability?Unit random processmodels of coins and uniform distributionHow to get the number of successes given $n$ trials , probability $P$ and a random variable $X$Absorbing Markov chain in a computer. Is “almost every” turned into always convergence in computer executions?Stopped random walk is not uniformly integrable